Find the value of t for at distribution with 40 degrees of freedom such that the area between-1 and equals 99 %. Round your answer to three decimal places, if nescarry

Answers

Answer 1

Answer:

The value is [tex]t = 2.705[/tex].

Step-by-step explanation:

In this question, we have to find the critical value for the t-distribution, with 40 degrees of freedom, and a 99% confidence level.

99% confidence level:

We have to find a value of T, which is found looking at the t table, with 40 degrees of freedom(y-axis) and a two-tailed value of [tex]1 - \frac{1 - 0.99}{2} = 0.995[/tex]. So we have T = 2.705.

The value is [tex]t = 2.705[/tex].


Related Questions

If two numbers differ by 9 the same of their squares is 653. What are the numbers?

Answers

Answer:

Two numbers differ by 9 and the sum of their square is 653. What are the numbers?

Well,that's a mathematical question from algebra and it's quite difficult to answer such questions by writing through the circumstances offered by apps like quora.

However,I have tried to answer your question in an understandable way.Hope you may not find it difficult to analyze.

Let the numbers be x and (9+x)

Therefore,according to given,

x^2 + (9+x)^2 =653

=>x^2 + (9)^2 + x^2 + 2×(9)×(x)=653 (Applying the formula of (a+b)^2)

=>x^2 + 81 + x^2 + 18x =653

=>2x^2 + 18x + (81-653)=0

=>2x^2 + 18x - 572=0

=>2x^2 + (44x - 26x) - 572=0

=>2x^2 + 44x - 26x - 572=0

=>2x(x + 22) - 26(x + 22)=0

=>(x + 22)(2x - 26)=0

But since the number can't be negative

Therefore, x=13

Hence,the required numbers are 13 and 22.

Step-by-step explanation:

in first hope you like it

Martha, Lee, Nancy, Paul, and Armando have all been invited to a dinner party. They arrive randomly, and each person arrives at a different time.

a. In how many ways can they arrive?

b. In how many ways can Martha arrive first and Armando last?

c. Find the probability that Martha will arrive first and Armando last.

Show your work

Answers

Answer:

a) 120

b) 6

c) 1/20

Step-by-step explanation:

a) 5! = 120

b) (5 - 2)! = 6

c) 6/120 = 1/20

(SAT PREP) Find the value of x in each of the following excersises

Answers

Answer:

The answer is 155.

Step-by-step explanation:

We can find the remaining parts of the triangle angles.

a man spends RS 608 a month. If he earns Rs 640, what percentage of his invome does he save??.


Please explanation​

Answers

Answer:

5%

Step-by-step explanation:

given,

Earns= Rs 640

spends= Rs 608

saves= (Rs 640 - Rs 608)

=Rs 32

therefore, 32/640x100

answer = 5%

Answer From Gauth Math

Answer:

5%

Step-by-step explanation:

save=640-608=32

(32/640)*100%

5%

what Is the si unit of temperature​

Answers

Answer:

the Si unit of temprature in Kelvin (K)

Step-by-step explanation:

Answer:

The answer is Kelvin (k).

Step-by-step explanation:

The kelvin (K) is defined by taking the fixed numerical value of the Boltzmann constant k to be [tex]1.380649*10^{-23}[/tex] when expressed in the unit of joule per kelvin. The temperature 0 K is commonly referred to as "absolute zero." On the widely used Celsius temperature scale, water freezes at 0 °C and boils at about 100 °C. One Celsius degree is an interval of 1 K, and zero degrees Celsius is 273.15 K. An interval of one Celsius degree corresponds to an interval of 1.8 Fahrenheit degrees on the Fahrenheit temperature scale.

The kelvin is also the fundamental unit of the Kelvin scale, an absolute temperature scale named for the British physicist William Thomson (known as Lord Kelvin). An absolute temperature scale has as its zero point absolute zero (−273.15° on the Celsius temperature scale and −459.67° on the Fahrenheit temperature scale), the theoretical temperature at which the molecules of a substance have the lowest energy; hence, all values on such a scale are nonnegative.  

If side A is 10 inches long, and side B is 24 inches, find the length of the unknown side.

Answers

Step-by-step explanation:

Right Triangles and the Pythagorean Theorem. The Pythagorean Theorem, a2+b2=c2, a 2 + b 2 = c 2 , can be used to find the length of any side of a right triangle.

e lifetimes of lightbulbs of a particular type are normally distributed with a mean of290 hours and astandard deviation of6 hours. What percentage of the bulbs have lifetimes that lie within 1 standarddeviation to either side of the mean

Answers

Answer:

Step-by-step explanation:

[tex]p(\overline{X}-\sigma \leq X \leq \overline{X}+\sigma)\\\\=p(\dfrac{\overline{X}-\sigma -\overline{X} }{\sigma} \leq Z \leq \dfrac{\overline{X}+\sigma -\overline{X} }{\sigma} )\\\\=p ( -1 \leq Z \leq 1)\\\\=2*(\ p (Z \leq 1)-0.5)\\\\=2*(0.8413-0.5)\\\\=0.6826\\\\\approx{68\%}[/tex]

i need help with this question asapppppp

Answers

9514 1404 393

Answer:

  $11,680.58

Step-by-step explanation:

Usually, I would say copy the example, using 70,000 instead of 55,000. However, the example you show has a couple of errors in it. You need to do what it says, not follow what it did.

__

The first 48,535 is taxed at 15%, so the tax is 0.15×48535 = 7280.25.

The next (70,000 -48,535) = 21,465 is taxed at 20.5%, so the tax is ...

  0.205×21,465 = 4400.325 ≈ 4400.33

The the total tax due on $70,000 is ...

  $7280.25 +4400.33 = $11,680.58 . . . . tax due on $70,000

_____

Additional comments

The example shown has a couple of errors. The tax on the excess amount is figured at 2.05%, not 20.5%, and the 132.53 value from that is shown as 132.23.

__

Any tax table like this one can be reduced to a set of simpler formulas. Here are the formulas for the brackets shown in your tax table.

  ≤ 48535 -- income × 0.15

  ≤ 97069 -- income × 0.205 -2669.425

  ≤ 150,473 -- income × 0.26 -8008.22

  ≤ 214,368 -- income × 0.29 -12,522.41

  > 214,368 -- income × 0.33 -21,097.13

In this case, the second row of this simpler table would give the tax on $70,000 as ...

  tax = 70,000 × 0.205 -2669.425

  tax = 14350 -2669.425 = 11680.575 ≈ 11,680.58 . . . same as above

NO LINKS OR ANSWERING QUESTIONS YOU DON'T KNOW!!!

Chapter 11 part 2:

What are three different properties of logarithmic functions when encountering the operations of addition, subtraction, and multiplication? Provide an example of each.

Answers

The three main log rules you'll encounter are

log(A*B) = log(A) + log(B)log(A/B) = log(A) - log(B)log(A^B) = B*log(A)

The first rule allows us to go from a log of some product, to a sum of two logs. In short, we go from product to sum. The second rule allows us to go from a quotient to a difference. Lastly, the third rule allows to go from an exponential to a product.

Here are examples of each rule being used (in the exact order they were given earlier).

log(2*3) = log(2) + log(3)log(5/8) = log(5) - log(8)log(7^4) = 4*log(7)

----------------

Here's a slightly more complicated example where the log rules are used.

log(x^2y/z)

log(x^2y) - log(z)

log(x^2) + log(y) - log(z)

2*log(x) + log(y) - log(z)

Hopefully you can see which rules are being used for any given step. If not, then let me know and I'll go into more detail.

The diameters of ball bearings are distributed normally. The mean diameter is 7373 millimeters and the variance is 44. Find the probability that the diameter of a selected bearing is less than 7676 millimeters. Round your answer to four decimal places.

Answers

Answer:

0.9332

Step-by-step explanation:

We are given that

Mean diameter, [tex]\mu=73[/tex]

Variance, [tex]\sigma^2=4[/tex]

We have to find the probability that the diameter of a selected bearing is less than 76.

Standard deviation, [tex]\sigma=\sqrt{variance}=\sqrt{4}=2[/tex]

[tex]P(x<76)=P(\frac{x-\mu}{\sigma}<\frac{76-73}{2})[/tex]

[tex]P(x<76)=P(Z<\frac{3}{2})[/tex]

Where [tex]Z=\frac{x-\mu}{\sigma}[/tex]

[tex]P(x<76)=P(Z<1.5)[/tex]

[tex]P(x<76)=0.9332[/tex]

Hence, the probability that the diameter of a selected bearing is less than 76=0.9332

Help me please and thank you

Answers

Answer:

Option C is correct

Step-by-step explanation:

[tex]log( {10}^{3} )[/tex]

Use logarithm rules to move 3 out of the exponent.

[tex]3 \: log \: (10)[/tex]

Logarithm base 10 of 10 is 1.

[tex]3×1[/tex]

Multiply 3 by 1.

[tex]3[/tex]

Hope it is helpful....
C is the correct answer

I need help with ged

Answers

Answer:

General Educational Development (GED) tests

What do subject do you need help?

Step-by-step explanation:

The GED® exam is made up of 4 subjects, broken into separate exams: Mathematical Reasoning, Reasoning Through Language Arts, Social Studies, and Science.

I’m new to this app and I need help with those two questions please help!!

Answers

y=x²-10x-7

a>0 so we will be looking for minimum

x=-b/2a=10/2=5

y=25-50-7=-32

Answer: (5;32)

y=-4x²-8x+1

а<0 so we will be looking for maximum

х=-b/2a=8/-8=-1

у=4+8+1=13

Maximum point (-1;13)

Your car can go 2/7 of the way on 3/8 of a tank of gas how far can you go on the remaining gas?
A proportion that can be used is a/b=c/d

Answers

Answer:

10/21 of the distance

Step-by-step explanation:

2/7 distance

------------------

3/8 tank

The rest of the tank is 8/8 - 3/8 = 5/8

2/7 distance         x

------------------ = ----------------------

3/8 tank              5/8 tank

Using cross products

2/7 * 5/8 = 3/8x

10/56 = 3/8x

Multiply each side by 8/3

10/56 * 8/3 = 3/8x * 8/3

10/3 * 8/56=x

10/3 * 1/7 =x

10/21 =x

10/21 of the distance

the graph of f(x)=6(.25)^x and its reflection across the y-axis , g(x), are shown. what is the domain of g(x)

Answers

9514 1404 393

Answer:

  all real numbers

Step-by-step explanation:

The domain of any exponential function is "all real numbers". Reflecting the graph across the y-axis, by replacing x by -x does not change that.

The domain of g(x) = f(-x) is all real numbers.

someone help me pls i need to pass summer school

Answers

16 to the 0 power 16 to the 0 power 16 to the 0 power 16 to the 0 power 16 to the 0 power

Answer:

A

Step-by-step explanation:

The be the inverse function the domain {4,5,6,7} becomes the range and the range {14,12,10,8} becomes the domain

14 → 4

12 →5

10 →6

8 →7

3 x {(300 - 70 ÷ 5) - [3 x 23 - (8 - 2 x 3)]}

A.657
B.2433
C. -843

Answers

Answer:

657

Step-by-step explanation:

pemdas

The value of the expression 3 x {(300 - 70 ÷ 5) - [3 x 23 - (8 - 2 x 3)]} is 657.

Hence option A is correct.

Given is an expression, 3 x {(300 - 70 ÷ 5) - [3 x 23 - (8 - 2 x 3)]}, we need to simplify it,

Let's break down the expression step by step:

First, let's simplify the expression inside the innermost parentheses:

8 - 2 x 3 = 8 - 6 = 2

Next, let's simplify the expression inside the brackets:

3 x 23 - 2 = 69 - 2 = 67

Now, let's substitute the simplified expression inside the brackets back into the original expression:

(300 - 70 ÷ 5) - 67

Next, let's simplify the expression inside the remaining parentheses:

70 ÷ 5 = 14

Now, let's substitute the simplified expression inside the parentheses back into the expression:

(300 - 14) - 67

Next, let's simplify the expression inside the remaining parentheses:

300 - 14 = 286

Now, let's substitute the simplified expression inside the parentheses back into the expression:

286 - 67

Finally, let's perform the subtraction:

286 - 67 = 219

Now, let's multiply the result by 3:

3 x 219 = 657

Therefore, the value of the expression 3 x {(300 - 70 ÷ 5) - [3 x 23 - (8 - 2 x 3)]} is 657.

Learn more about expression click;

https://brainly.com/question/28170201

#SPJ2

find the derivative of y=(x³-5)⁴(x⁴+3)⁵ ​

Answers

Answer:

[tex]12x^{2} (x^{3}-5)^{3} (x^{4}+3)^{5} +20x^{3} (x^{3}-5)^{4} (x^{4}+3)^{4}[/tex]

Step-by-step explanation:

Explain why the equation x=x+1 is a contradiction

Answers

Answer:

It results in no solution.

Step-by-step explanation:

If you subtract x on both sides, this will leave you with 0 ≠ 3. The result is no solution. This is why it is contradictory.

Algebra II Part 1
Choose the expression or equation that correctly represents this information
Rose works eight hours a day for five days a week. How many hours will she work in sa
weeks?
hours = 40 = 6
hours = 40.6
hours = 6 = 40

Answers

Answer:

240 i.e 40*6

Step-by-step explanation:

if rose works 8hrs per day then she works 40 hrs per week (5 days) therefore 40 hrs per 6 weeks =40*6=240

Answer:

40

Step-by-step explanation:


I need help answering this ASAP

Answers

Answer:

A the input x=3 goes to two different output values

Step-by-step explanation:

This is not a function

x = 3 goes to two different y values

x = 3 goes to t = 10 and y = 5

Complete the equation describing how x
and y are related.
Х у
y = [? ]x +
07
1 9
2 11
3 13
4 15
5 17
Enter the answer that
belongs in [?]

Answers

Answer:

Hello,

Answer 2

Step-by-step explanation:

7=2*0+7

9=2*1+7

11=2*2+7

13=2*3+7

15=2*4+7

17=2*5+7

y=2*x+7

An other way:

[tex]points\ ( 0,7)\ and\ (1,9)\\\\\Delta\ y=9-7=2\\\Delta\ x=1-0=1\\\\\\y-7=(x-0)*2\\\\y=2x+7\\[/tex]

The complete equation is [tex]y = 2x+7[/tex].

What is equation?

An equation is a condition on a variable such that two expressions in the variable should have equal value.

What is substitution?

Substitution means replacing the variables (letters) in an algebraic expression with their numerical values.

According to the question.

We have a table which shows the relation between x and y.

Let the missing term be a and b.

The the given equation becomes

[tex]y = ax + b[/tex]

For finding the value of a and b.

Substitute x = 0 and y = 7 in equation y = ax + b.

[tex]\implies 7 = a(0) + b\\\implies b = 7[/tex]

Again, substitute  x = 1 and y = 9 in the equation y = ax+ b

[tex]\implies 9 = a(1) +b\\\implies 9 = a + 7\\\implies a = 2[/tex]

substitute the value of a and b in the equation y = ax + b.

[tex]\implies y = 2x+ 7[/tex]

Therefore, the complete equation is [tex]y = 2x+7[/tex].

Find out more information about equation and substitution here:

https://brainly.com/question/2581775

#SPJ2

596 is divisible by 2?
a.yes
b.no

Answers

Answer:

It's yes

Step-by-step explanation:

Answer:

yep

Step-by-step explanation:

number is even, so it can be evenly divided by 2. :)

What is the equation of the line that passes through (4,3) and (2, -1)?

y = 4x -13
y = 6x+4
y = 2x-5
y = 1/2 x -2

Answers

Answer:

y=2x-5

Step-by-step explanation:

By using two-points form:

y-y1/y2-y1=x-x1/x2-x1

p(x1,y1)=(4,3)

p(x2,y2)=(2,-1)

Subtitute points in formula:

y-3/-1-3=x-4/2-4

y-3/-4=x-4/-2

y-3/-2=x-4/-1

1(y-3)=2(x-4)

y-3=2x-8

y=2x-8+3

y=2x-5

Note:if you need to ask any question please let me know.

After running 3/4 of a mile tess has only run 1/3 how long is the race in miles but I want to know how you did it

Answers

Take 3/4 and 1/3 multiply the bottom, thats your denominator. Then since you multiplied 4•3, multiply the top by three.

One angle of a triangle is equal to the sum of the remaining angles. If the ratio of measures of the ren
is 2:1, find the measures of the three angles of the triangle.

Answers

9514 1404 393

Answer:

  90°, 60°, 30°

Step-by-step explanation:

The remaining angles have a ratio of 2:1, so total 3 "ratio units". The first angle is equal to that sum: 3 ratio units, so all of the angles together total 3+2+1 = 6 ratio units. The total of angles is 180°, so each ratio unit is 180°/6 = 30°.

The first angle is 3 ratio units, or 90°.

The second angle is 2 ratio units, or 60°.

The third angle is half that, or 30°.

The three angles are 90°, 60°, 30°.

Solving Equations by Dividing 2) 9x= -135 Solve for x. 0 -144 O 126 O 15 0 -15

Answers

X = 15
Explanation:
9x = -135
Divide by 9 on both sides
X = -15
It is -15 since 135 is negative

[tex]\huge\text{Hey there!}[/tex]

[tex]\huge\boxed{\mathsf{9x = -135}}[/tex]

[tex]\huge\boxed{\text{DIVIDE 9 to BOTH SIDES}}[/tex]

[tex]\huge\boxed{\mathsf{\dfrac{9x}{9}= \dfrac{-135}{9}}}[/tex]

[tex]\huge\boxed{\mathsf{\bullet \ CANCEL: \dfrac{9}{9}\ because\ it \ gives\ you\ 1}}[/tex]

[tex]\huge\boxed{\bullet\ \mathsf{KEEP: \dfrac{-135}{9}\ because\ it\ helps\ solve \ for}}\\\huge\boxed{\mathsf{the\ x-value}}[/tex]

[tex]\huge\boxed{\mathsf{x = \dfrac{-135}{9}}}\\\\\huge\boxed{\mathsf{\dfrac{-135}{9}= x}}}[/tex]

[tex]\huge\boxed{Simplify \ it\uparrow}[/tex]

[tex]\huge\boxed{\mathsf{x = \bf -15}}[/tex]

[tex]\huge\boxed{\textsf{Therefore, your answer is: Option D. -15 }}\huge\checkmark[/tex]

[tex]\huge\text{Good luck on your assignment \& enjoy your day!}[/tex]

~[tex]\huge\boxed{\frak{Amphitrite1040:)}}[/tex]

help asap!!
Find the length of AB
A. 2.89
B. 33.13
C. 378.63
D. 377.19

Answers

Answer:

C

Step-by-step explanation:

[tex] \sin( 5 ^{o} ) = \frac{33}{ab} \\ ab = 378.63[/tex]

The answer is C……………….

factorise completely 4x^(2 )(x + 1) - 6x (x+1)

Answers

Answer:

[tex] {4x}^{2} (x + 1) - 6x(x + 1) \\ = (x + 1)(4 {x}^{2} - 6 x ) \\ = (x + 1)(2x)(2x - 3)[/tex]

explanation:

first choose the common factor by observation, it is (x + 1):

factorise it out:

= (x + 1)(4x² - 6x)

by observation in (4x² - 6x), common factor is 2x.

Factorise 2x out:

= (x + 1)[2x(2x - 3)]

Answer:

(4x2-6x) (x+1)

now common factor is (x+1) ,so,(4x2-6x) (x+1)

Smart phone: Among 239 cell phone owners aged 18-24 surveyed, 103 said their phone was an Android phone. Part: 0 / 30 of 3 Parts Complete Part 1 of 3 (a) Find a point estimate for the proportion of cell phone owners aged 18-24 who have an Android phone. Round the answer to at least three decimal places. The point estimate for the proportion of cell phone owners aged 18-24 who have an Android phone is .

Answers

Answer:

The point estimate for the proportion of cell phone owners aged 18-24 who have an Android phone is 0.4137.

Step-by-step explanation:

The point estimate is the sample proportion.

Sample proportion:

103 out of 249, so:

[tex]p = \frac{103}{249} = 0.4137[/tex]

The point estimate for the proportion of cell phone owners aged 18-24 who have an Android phone is 0.4137.

Other Questions
What is the value of the expression (2x + y) (2x - y) when x = 4 and y = -5? Arya has a homeowners policy that covers up to $400,000. Her house is completely destroyed in a fire. When an assessment is made of the house and possessions, it is decided that her losses are only worth about $350,000. How much money will Arya receive from her insurer (assuming there is no deductible)? A. $50,000 B. $200,000 C. $350,000 D. $400,000 Cul es su punto de vista sobre llevar mascarilla para todos lados? A uniform plastic block floats in water with 50.0 % of its volume above the surface of the water. The block is placed in a second liquid and floats with 23.0 % of its volume above the surface of the liquid.What is the density of the second liquid?Express your answer with the appropriate units. A ball is dropped from a height of 9 feet and allowed to bounce. Each time the ball bounces, it reaches a height that is23the height of the previous bounce. How many times will the ball be exactly 3 feet above the ground? The image shows the MyPlate tool.A plate showing food groups. In order from greatest amount to smallest amount, the food groups are protein, fruits, vegetables, grains, and dairy.Nadeem is gathering information to improve a diet plan. What tips should he use from the MyPlate daily food plan? Select three options.Choose whole grains.Eat a variety of fruits.Add in low-fat dairy products.Eat only green vegetables.Choose high-fat protein varieties. Which of the following is the point and slope of the equation y + 14 = 7(x - 18)? List three main rhetorical devices used in persuasive essays Find the range from the ordered pair {(1, 2), (2, 3), (3, 4), (4, 5)} solve for x. round to the nearest tenth of a degree, if necessary. 57 117find x triangle Compared with dim light, what do light waves that look bright tend to have the subject just says science but the picker doesn't have that what do you think the need and importance of a constitution? nguyn tc xc lp li tc tin vay What is the solution to the inequality -6+|2p+3| > 7 Suppose an economy starts the year with $100 million in capital, and during the course of a year, it adds $20 million of gross investment. Economists estimate that the depreciation rate for this economy is 9% per year.Required:a. Calculate depreciation and net investment for this economy.b. Calculate the value of net exports. HELPPP PLEASE ASAP!!! I dont know how to solve this problem nor where to start? Can some please help me out and explain how you got the answer please. Thank you for your time. Can someone please help me? HELPPPP what is this An organization's budget program should not be used: Group of answer choices to motivate employees. to assign blame to managers that do not meet budgetary goals. to help evaluate managers. to allocate resources to the various parts of an organization.